\DocumentMetadata{} \documentclass[dvipsnames,12pt]{exam} % compiles with: latexmk -pdflatex=lualatex -pdf exam.tex \usepackage[top = 2cm, bottom = 3cm, left=1.5cm, right=1.5cm]{geometry} \usepackage{microtype,fontspec,amssymb,titlesec,multicol,braket,graphicx,tikz,pgfplots} \usepgfplotslibrary{fillbetween} \usetikzlibrary{intersections,arrows.meta,decorations.pathreplacing} \graphicspath{{./img/}} \usepackage[shortlabels]{enumitem} \usepackage{xcolor,cancel,amsmath,hyperref,eso-pic} \runningfooter{}{}{\thepage} \runningheader{}{}{\scriptsize Aayush Bajaj | z5362216} \hypersetup{ colorlinks = true, linkcolor = RedViolet, citecolor = gray } \titleformat{\section}{\normalfont\Large\bfseries}{{\color{RedViolet}\S}}{0.5em}{} \titleformat{\subsection}{\normalfont\large\bfseries}{{\large \color{RedViolet}\S\S}}{0.5em}{} \titleformat{\subsubsection}{\normalfont\bfseries}{{\color{RedViolet}\S\S\S}}{0.5em}{} \parindent 0pt %%% defs \newcommand{\N}{\mathbb{N}} \newcommand{\C}{\mathbb{C}} \newcommand{\D}{\mathbb{D}} \newcommand{\F}{\mathcal{F}} \renewcommand{\P}{\mathcal{P}} %careful \newcommand{\R}{\mathbb{R}} \newcommand{\Q}{\mathbb{Q}} \newcommand{\T}{\mathbb{T}} \newcommand{\Z}{\mathbb{Z}} \newcommand{\ds}{\displaystyle} \newcommand{\st}{\,:\,} \newcommand{\norm}[1]{\Vert #1 \Vert} \newcommand{\cl}{\mathop{\mathrm{cl}}} \newcommand{\Int}{\mathop{\mathrm{Int}}} %%% end defs \usepackage{amsthm} \newtheorem{theorem}{Theorem}[section] \newtheorem{lemma}[theorem]{Lemma} \newtheorem{prop}{Proposition} \newtheorem{remark}{Remark} \newtheorem{definition}{Definition}[section] \AddToShipoutPictureBG*{ \AtPageCenter{\makebox(0,0){\rotatebox{45}{\textcolor{gray!30}{\fontsize{70}{70}\selectfont Property of MATH3611}}}} } \author{Aayush Bajaj | z5362216} \date{\today} \title{MATH3611 --- Final Solutions} % Toggle solutions: \printanswers %\noprintanswers \begin{document} \maketitle \dotfill \tableofcontents \vspace{0.5cm} \newpage %%%%%%%%%%%%%%%%%%%%%%%%%%%%%%%%%%%%%%%%%%%%%%%%%%%%%%%%%%%%%%%%%%%%%%%%%%%%%%%% % QUESTIONS %%%%%%%%%%%%%%%%%%%%%%%%%%%%%%%%%%%%%%%%%%%%%%%%%%%%%%%%%%%%%%%%%%%%%%%%%%%%%%%% \begin{questions} %=============================================================================== \addcontentsline{toc}{section}{Q1. Set Theory} \question[15] Set Theory \begin{parts} \part[5] \begin{subparts} \subpart $|A|\le |B|$ \begin{solution}means there exists an \textbf{injective} map $f:A\to B$.\end{solution} \subpart $|A|=|B|$ \begin{solution}means there exists a \textbf{bijective} map $f:A\to B$.\end{solution} \subpart $|A|<|B|$ \begin{solution}means $\exists$ injective but not \textbf{surjective} map $A\to B$.\end{solution} \subpart Prove that $|\N| = |2\N|$. \begin{solution} Consider $f:\N\to 2\N$ given by $f(n)=2n$. It is clearly injective and surjective. The correspondence is illustrated below. \begin{center} \begin{tikzpicture}[>=Latex, node distance=1.6cm] % top row: N \node (nlabel) at (-1.0,0) {$\N:$}; \node (n0) at (0,0) {$0$}; \node (n1) at (1.5,0) {$1$}; \node (n2) at (3.0,0) {$2$}; \node (n3) at (4.5,0) {$3$}; \node (ndots) at (6.0,0) {$\cdots$}; % bottom row: 2N \node (elabel) at (-1.0,-1.6) {$2\N:$}; \node (e0) at (0,-1.6) {$0$}; \node (e1) at (1.5,-1.6) {$2$}; \node (e2) at (3.0,-1.6) {$4$}; \node (e3) at (4.5,-1.6) {$6$}; \node (edots) at (6.0,-1.6) {$\cdots$}; % vertical “ladder” arrows (double-lined head via Implies) \draw[-{Implies[length=7pt]}] (n0) -- (e0); \draw[-{Implies[length=7pt]}] (n1) -- (e1); \draw[-{Implies[length=7pt]}] (n2) -- (e2); \draw[-{Implies[length=7pt]}] (n3) -- (e3); % braces for labels \draw[decorate,decoration={brace,amplitude=5pt}] (-0.6,0.5) -- (6.6,0.5) node[midway, yshift=10pt]{top row: domain}; \draw[decorate,decoration={brace,mirror,amplitude=5pt}] (-0.6,-2.1) -- (6.6,-2.1) node[midway, yshift=-10pt]{bottom row: codomain}; \end{tikzpicture} \end{center} Thus $f$ is a bijection and $|\N|=|2\N|$. \end{solution} \end{subparts} \part[10] \begin{subparts} \subpart State the Schroder--Bernstein Theorem. This is also known as the Cantor Bernstein Theorem. \begin{solution} If $|A|\le|B|$ and $|B|\le|A|$ then $|A|=|B|$. \end{solution} \subpart If $A$ is infinite, show $|\N|\le|A|$. \begin{solution} Pick distinct $a_0,a_1,\dots\in A$ recursively; $n\mapsto a_n$ is injective $\N\hookrightarrow A$. \end{solution} \subpart Deduce $|A\cup\N|=|A|$ for infinite $A$. \begin{solution} Trivially $|A|\le|A\cup\N|$. From (b) get injection $\N\hookrightarrow A$; combine with inclusion $A\hookrightarrow A$ to build an injection $A\cup\N\hookrightarrow A$ by sending $n\mapsto a_{2n+1}$ and $a_k\mapsto a_{2k}$. Apply Schröder--Bernstein. \end{solution} \subpart If $A$ is countably infinite prove that $|\N| \leq |A|$. \begin{solution} Since $A$ is countably infinite, there exists a bijection $h:\N\to A$, hence a fortiori an injection $\N\hookrightarrow A$. Therefore $|\N|\le |A|$. \end{solution} \end{subparts} \end{parts} %=============================================================================== \addcontentsline{toc}{section}{Q2. Metric Spaces} \question[13] Metric Spaces \begin{parts} \part Define a Metric Space $(X,d)$. \begin{solution} A metric space is a set $X$ with $d:X\times X\to[0,\infty)$ such that for all $x,y,z\in X$: \[ d(x,y)=0\iff x=y,\quad d(x,y)=d(y,x),\quad d(x,z)\le d(x,y)+d(y,z). \] \end{solution} \part Define an open set $Y\subseteq X$. \begin{solution} $U\subseteq X$ is open if for each $x\in U$ there exists $r>0$ with the open ball $B(x,r)=\{y\in X: d(x,y)0,\ B(x,r)\subseteq Y\,\}, \] a union of open balls. Unions of open sets are open, so $\Int(Y)$ is open. \end{solution} \end{parts} %=============================================================================== \addcontentsline{toc}{section}{Q3. Sequences} \question[5] Suppose $\limsup x_n=a$ and $\limsup x_n=b$. Prove $a=b$. \begin{solution} Assume $ab-\varepsilon$. Thus for some $n$, \[ b-\varepsilon0$ $\exists N$ s.t.\ $\|f_n-f_m\|_\infty<\varepsilon$ for $m,n\ge N$. Fix $x\in X$. Then \[ |f_n(x)-f_m(x)|\le \|f_n-f_m\|_\infty<\varepsilon\quad(m,n\ge N), \] so $(f_n(x))$ is Cauchy in $\R$ (or $\C$) and hence convergent. Define $f(x)=\lim_{n\to\infty} f_n(x)$. Thus $f_n\to f$ pointwise. \end{solution} \part Hence or otherwise prove that the limit $f$ is continuous (under the standard hypothesis). \begin{solution} If each $f_n$ is \emph{continuous} and $f_n\to f$ in $\|\cdot\|_\infty$ (i.e.\ uniformly), then $f$ is continuous as a uniform limit of continuous functions. (No compactness assumption is needed for this implication.) \end{solution} \part Show $c_{00}$ with the $\ell_1$ metric is not complete. \begin{solution} Let $x^{(n)}=(1,1/2,\ldots,1/2^{\,n-1},0,0,\ldots)\in c_{00}$. For $m>n$, \[ \|x^{(m)}-x^{(n)}\|_1=\sum_{k=n}^{m-1}2^{-k}\le 2^{-(n-1)}\xrightarrow{n\to\infty}0, \] so $(x^{(n)})$ is Cauchy. In $\ell^1$, $x^{(n)}\to x=(1,1/2,1/4,\ldots)$, but $x\notin c_{00}$. Hence $c_{00}$ is not complete. \end{solution} \end{parts} %=============================================================================== \addcontentsline{toc}{section}{Q5. Topology, Compactness} \question[11] Topology, Compactness \begin{parts} \part Define a Hausdorff Space. \begin{solution} $(X,\tau)$ is Hausdorff if for all $x\neq y$ there exist disjoint $U,V\in\tau$ with $x\in U$, $y\in V$. \end{solution} \part Define a compact space. \begin{solution} $(X,\tau)$ is compact if every open cover admits a finite subcover. \end{solution} \part Consider \[ \tau=\{\varnothing,\R\}\ \cup\ \{(-t,t)\subset\R:\ t>0\}. \] \begin{subparts} \subpart Define a topology. \begin{solution} A topology $\tau$ on $X$ is a collection of subsets of $X$ containing $\varnothing$ and $X$, closed under arbitrary unions and finite intersections. Members of $\tau$ are the open sets. \end{solution} \subpart Prove $\tau$ is a topology on $\R$. \begin{solution} $\varnothing,\R\in\tau$ by definition. Arbitrary unions: a union of sets $(-t_i,t_i)$ is either $\R$ (if $t_i$ unbounded) or $(-T,T)$ with $T=\sup_i t_i$; both in $\tau$, and unions with $\R$ give $\R$. Finite intersections: $(-s,s)\cap(-t,t)=(-\min\{s,t\},\min\{s,t\})\in\tau$, and intersections with $\R$ return the other set. Hence $\tau$ is a topology. \end{solution} \subpart Find the limit(s) of the sequence $x_n=(-1)^n$ in $(\R,\tau)$. \begin{solution} Nontrivial basic neighborhoods are $(-t,t)$ about $0$. For $y\neq 0$, the only open set containing $y$ is $\R$, so the neighborhood condition is vacuous and \emph{every} sequence converges to $y$. For $0$, neighborhoods are $(-t,t)$; since $(-1)^n\notin(-t,t)$ for $t<1$, the sequence is not eventually in any neighborhood of $0$. Therefore $(-1)^n$ converges to every $y\in\R\setminus\{0\}$ and to no other point. \end{solution} \end{subparts} \part Let $X$ be Hausdorff and $Y\subseteq X$ compact. Prove $Y$ is closed in $X$. \begin{solution} For $x\in X\setminus Y$ and each $y\in Y$, choose disjoint opens $U_y\ni x$, $V_y\ni y$. The $\{V_y\}_{y\in Y}$ cover $Y$, so compactness yields $y_1,\dots,y_k$ with $Y\subset\bigcup_{i=1}^k V_{y_i}$. Then $U=\bigcap_{i=1}^k U_{y_i}$ is an open neighborhood of $x$ disjoint from $Y$. Hence $X\setminus Y$ is open, so $Y$ is closed. \end{solution} \end{parts} \end{questions} \end{document}